PTC.S3.Q2 - Mount Shalko is the perfect site for the proposed...

rowdyhouseplantsrowdyhouseplants Core Member
edited August 2023 in Logical Reasoning 21 karma

"Astronomer: Mount Shalko is the perfect site for the proposed...."

Can someone please explain why C is the correct answer? I didn't like any of the answer choices but ended up picking E (even though I was so sure this was wrong, it seemed like the best choice at the time)

Comments

  • luffyyyyluffyyyy Alum Member
    698 karma

    Basically the stimulus points out all the advantages / benefits of picking Mount Shalko as the site for the proposed observatory. One of the premises used to support the astronomer's argument is that the observatory would not be a threat to endemic life forms because it would preclude recreational use of the mountain, which means that 200,000 recreational users wouldn't be able to visit the mountain. The stimulus tells us recreational use poses a threat to the wildlife. So the stimulus seems to give us another reason why precluding recreational use is great - wildlife wouldn't be threatened. But C tells us that the complex that's designed to be used could be just as disruptive to the ecology as the current level of recreational use. This weakens the support because if the complex could disrupt the life forms just as much as the recreational use, then it doesn't seem like a benefit / advantage anymore. I hope this helps!

  • rowdyhouseplantsrowdyhouseplants Core Member
    21 karma

    this helped, thank you! i think the message of "accepting all of the premises in the stimulus as they are" in earlier lessons is preventing me from picking the right choice. i must have overlooked C because i thought it was undermining this premise. but i'm noticing that Weakening questions often require you to challenge the premises and their validity

Sign In or Register to comment.